tzyc
Thanks Received: 0
Atticus Finch
Atticus Finch
 
Posts: 323
Joined: May 27th, 2012
 
 
trophy
Most Thankful
 

Q12 - One year ago a locak

by tzyc Mon Feb 18, 2013 7:47 am

In the stimulus, it says; because of the advertisement, the number of smoker decreased...
So I chose (B) to show it actually decreased. But maybe "substantial number" is too strong so it's wrong?
Also I'm not sure why (D) is correct...at first I even thought it could weaken the argument :|
 
sumukh09
Thanks Received: 139
Atticus Finch
Atticus Finch
 
Posts: 327
Joined: June 03rd, 2012
 
 
trophy
Most Thanked
trophy
First Responder
 

Re: Q12 - One year ago a locak

by sumukh09 Mon Feb 18, 2013 5:31 pm

I would argue B is wrong because the people in the locality still smoked cigarettes, albeit in less of a quantity - but that doesn't strengthen the argument since the campaign's purpose was to stop people from smoking completely

D is correct because it shows how the campaign had a direct effect on people. If the merchants reduced the price of cigarettes by the same amount of the tax, then the price of cigarettes would be unchanged and so there would not be a economic reason not to smoke cigarettes since the price is not any more expensive even with the tax. Clearly this demonstrates the impact that the campaign itself had on the locality as more people are now seemingly more aware of the detriments of smoking.
 
tzyc
Thanks Received: 0
Atticus Finch
Atticus Finch
 
Posts: 323
Joined: May 27th, 2012
 
 
trophy
Most Thankful
 

Re: Q12 - One year ago a locak

by tzyc Mon Feb 18, 2013 7:14 pm

Hi, thanks for your reply.
But I'm still confused...
If the price does not change, the consumer would continue to buy cigarettes, wouldn't they?
So I thought the number of purchase does not decrease...
Or do you mean the fact that the merchant need to change the price shows the impact of the campaign?
 
sumukh09
Thanks Received: 139
Atticus Finch
Atticus Finch
 
Posts: 327
Joined: June 03rd, 2012
 
This post thanked 1 time.
 
trophy
Most Thanked
trophy
First Responder
 

Re: Q12 - One year ago a locak

by sumukh09 Mon Feb 18, 2013 7:50 pm

tz_strawberry Wrote:Or do you mean the fact that the merchant need to change the price shows the impact of the campaign?


Exactly.

The merchant responded to the imposition of the tax by reducing the sale price of cigarettes by the same amount of the tax ie) the price of cigarettes is the same as before the campaign. So now there's no way to make the argument that people are buying less cigarettes because of the tax.. since the tax was offset by the reduction in the price of cigarettes; but rather the reason people stopped smoking was more likely because of the impact the campaign had and this strengthen's the argument's conclusion.
 
Nina
Thanks Received: 0
Atticus Finch
Atticus Finch
 
Posts: 103
Joined: October 15th, 2012
 
 
 

Re: Q12 - One year ago a locak

by Nina Thu Sep 26, 2013 5:26 pm

and how about answer A? i feel like A also eliminates an alternative reason, i.e. people are not increasingly buying other tobacco products for substituting smoking. which means the advertisements did have an effect in reducing the number of people smoking.

any help will be appreciated!
 
sumukh09
Thanks Received: 139
Atticus Finch
Atticus Finch
 
Posts: 327
Joined: June 03rd, 2012
 
This post thanked 1 time.
 
trophy
Most Thanked
trophy
First Responder
 

Re: Q12 - One year ago a locak

by sumukh09 Thu Sep 26, 2013 8:10 pm

Hi Nina,

Here's what I thought when I read A: "oh, residents haven't increased use of other harmful things like cigarettes since the ad.. so what, though? How does that help me strengthen the argument that people smoked less because of the advertisements?"

Nina Wrote: i feel like A also eliminates an alternative reason, i.e. people are not increasingly buying other tobacco products for substituting smoking. which means the advertisements did have an effect in reducing the number of people smoking.


What if they are buying the same amount of snuff/chewing tobacco as before but not necessarily increasing their usage of them? The ad, then, clearly did not have an effect. Just because they haven't increased the use doesn't mean they were less inclined to buy these products since the publication of the ad. When you're increasingly doing something, you are doing it in greater amounts; A says they are not doing these things in greater amounts, but they could still be chewing tobacco at the same rate as before the ad; which would imply the ad did not deter them from other tobacco products. Also, the ad was about cigarettes; sure tobacco is the reason cigarettes are so bad, but tobacco smoke is different than chewing tobacco or snuff - or at least I would imagine there is a difference. Basically, I wouldn't feel safe in making the assumption that the ad targeted the harmfulness of tobacco for the purpose of educating readers about the dangers of smoking. There could have been other reasons the ad used for making that illustration - not necessarily attacking tobacco.
User avatar
 
tommywallach
Thanks Received: 468
Atticus Finch
Atticus Finch
 
Posts: 1041
Joined: August 11th, 2009
 
This post thanked 1 time.
 
 

Re: Q12 - One year ago a local

by tommywallach Sat Sep 28, 2013 2:42 pm

Hey Guys,

Let's take this bottom to top. It's a strengthen, so we start with the core.

Conclusion: Ads had a small effect on people smoking cigarettes.
Premise: There was an ad, which was financed by a tax on cigarettes, and smoking went down a little.

The problem here is we don't know if the ad caused the change, or the increase in price!

(A) tells us that people aren't using other forms of tobacco. But this argument is only about cigarettes. Whether people are doing more of other things is irrelevant.

(B) is very tricky. This would actually imply that the effect was larger, but the conclusion says it should be quite small.

(C) is also tough. We don't know if this is related to smoking, or some other issue. It just says "respiratory ailments."

(D) CORRECT. This is the only answer choice that directly relates to the assumption we predicted. If merchants responded to the tax by lowering prices, cigarettes remained the same price. If sales still went down, it must have been because of the content of the ad.

(E) weakens the argument, because it means smokers are more likely to make a decision based on economic concerns.

Hope that helps!

-t


#officialexplanation
Tommy Wallach
Manhattan LSAT Instructor
twallach@manhattanprep.com
Image
 
Alvanith
Thanks Received: 0
Forum Guests
 
Posts: 25
Joined: October 20th, 2013
 
 
 

Re: Q12 - One year ago a locak

by Alvanith Mon Apr 07, 2014 7:17 am

I guess (B) is incorrect because the conclusion is the ads had an effect on the NUMBER of people who smoke cigarettes.

(B) just says a lot of people smoke less during the campaign, but does it strengthen the idea that the campaign has an effect on the numbers? Does it imply some people quite smoking? Hard to say, unless we put in some additional assumptions.

Stick to the core of the argument is what we need here...

Please correct me if I miss something here. Thanks.
User avatar
 
WaltGrace1983
Thanks Received: 207
Atticus Finch
Atticus Finch
 
Posts: 837
Joined: March 30th, 2013
 
 
trophy
Most Thanked
trophy
Most Thankful
trophy
First Responder
 

Re: Q12 - One year ago a locak

by WaltGrace1983 Tue Jun 24, 2014 9:17 pm

Alvanith Wrote:I guess (B) is incorrect because the conclusion is the ads had an effect on the NUMBER of people who smoke cigarettes.

(B) just says a lot of people smoke less during the campaign, but does it strengthen the idea that the campaign has an effect on the numbers? Does it imply some people quite smoking? Hard to say, unless we put in some additional assumptions.

Stick to the core of the argument is what we need here...

Please correct me if I miss something here. Thanks.


I was down to (B) and (D) too and so I reread the conclusion. It was focusing solely on "number" so I thought to myself, "well (B) is no good because it actually has nothing to do with the number who quite completely - which is what we are concerned about."

The reason why (B) is tricky is because it strengthens the idea that the advertisements had an effect. However, that is NOT the conclusion. The conclusion is that the advertisements had an effect...on the number of people who smoke cigarettes. Tricky, tricky, LSAT.

In addition, (B) is tricky because - without realizing that cigarettes were taxed by $.20, you would simply assume that (B) would actually weaken. After all, it seems to provide an alternative explanation as to why the number of people that smoked went down, a reason unrelated to the advertisements. However, the critical component to understand is that tax. Without realizing the effect of the tax, you are at risk of misunderstanding (B).

(D) basically eliminates an alternative explanation, the most "obvious" alternative explanation: price. You hear it all the time, "I want to stop smoking because I am tired of spending $ X.XX dollars per day on cigarettes." Well if the price is the same, as stated by (D), but LESS people are still buying cigarettes, this shows that maybe it was the advertisements after all.
 
kumsayuya
Thanks Received: 0
Vinny Gambini
Vinny Gambini
 
Posts: 17
Joined: June 07th, 2014
 
 
 

Re: Q12 - One year ago a locak

by kumsayuya Tue Jul 15, 2014 8:44 pm

Hi Walt, good explanation.

I think me and you had different reasons for getting rid of (B). I got rid of it pretty quickly by realizing that it states people smoke LESS, but this isn't that important is it?

I think where it is confusing in the stimulus is where it says "what was said in the advertisements had an effect.." and I think this is fair to say its referential phrasing to the 3% of the people in the population who quit. So therefore, even if they smoke LESS it doesn't really do much for the conclusion does it? Because smoking less doesn't necessarily result in a monetary change (represented in percentages in this question), but rather a frequency, but if that frequency were to be put into percent, we would be left in the same place (with the same percent of people being smokers) which makes me think its actually just sort of an irrelevant answer choice.

Anyone else have some thoughts on this?
 
christine.defenbaugh
Thanks Received: 585
Atticus Finch
Atticus Finch
 
Posts: 536
Joined: May 17th, 2013
 
This post thanked 1 time.
 
 

Re: Q12 - One year ago a locak

by christine.defenbaugh Tue Jul 22, 2014 2:17 pm

Great discussion, you two!

It actually sounds to me like you and WaltGrace1983 were focused on really similar things, kumsayuya.

The critical things to understand is what type of reduction is being discussed in both the premise and the conclusion of this argument.

The premise tells us that the 3% reduction is specifically in "the number of people in the locality who smoke cigarettes." The conclusion, again, refers to the effect on "the number of people in the locality who smoke cigarettes."

That means that a reduction in the number of cigarettes smoked by still-smokers has no bearing whatsoever on the argument, and it is an irrelevant answer, as you point out, kumsayuya.

I think that WaltGrace1983 may have meant that (D) sounds like a weakener at first, if you're not careful. If you read the conclusion as simply suggesting that the campaign (ads + tax) had an effect, (D) would seem to undermine that idea by suggesting that the tax didn't actually LAND on people.

To recognize (D) as a strengthener, you must realize that the conclusion isn't just about the whole *campaign* having an impact, but rather that it was the ad part of the campaign that made the difference, rather than the tax part of it. (D) removes any possible impact the tax could have had on the smokers, making it all the more likely that the impact came from the ad's content.

Keep up the great work guys!
 
af10
Thanks Received: 4
Vinny Gambini
Vinny Gambini
 
Posts: 11
Joined: June 30th, 2014
 
 
 

Re: Q12 - One year ago a locak

by af10 Sun Mar 01, 2015 3:16 am

I'm a bit dissatisfied with the explanations for why (A) is incorrect.

Completely understand why (D) is a correct answer, but also think (A) works, too.

Okay, so the first reason for the elimination of (A) was that they could be buying the same amount of snuff + chewing tobacco.

To that, I respond: The simple fact that it does NOT increase is sufficient. It could have stayed the same to the decimal or drastically decreased, but the knowing that it they didn't substitute their smoking fix for chewing tobacco/snuff is enough. Yes, the argument says they initiated an "anti-smoking advertising campaign" and that it was responsible for the decrease in those who "smoke cigarettes" ..that by no means means that (A) isn't out of scope or beyond the point.

Example.
If I put an ad up and it says "Stop eating [specific unhealthy food]" which was paid for by a tax from my dad. My dad stops eating the unhealthy food. How exactly would I know if it was my ad that did the trick? Well, for one, if we know that the prices somehow went back to their normal ones (say, my mom reimbursed him the tax) and he still didn't eat the unhealthy food, that'd help. However, if I know that he also didn't ..instead, buy extra deep fried double chocolate fudge Oreos or increase his fast food intake, that'd surely help too! The ad is about unhealthy foods, in which, if it is the case that his intake of those things increased, we can't for sure say it was my ad.

The ad/argument is based on cigarettes (DEFINITION: FINELY CUT TOBACCO - WIKIPEDIA) so it makes COMPLETE sense that ...hey, if they didn't get their fix elsewhere, that'd really help my case.

tommywallach Wrote:(A) tells us that people aren't using other forms of tobacco. But this argument is only about cigarettes. Whether people are doing more of other things is irrelevant. (And remember, nothing here mentions "replacement" of one tobacco product for another. You could be doing both (cigarettes + snuff) just as easily as only one).

-t


Exactly how is the fact that people are using other forms of tobacco irrelevant? This argument is essentially that their advertisement worked because people stopped smoking cigarettes. People don't smoke because smoke coming out of their mouth looks cool. If they're replacing the tobacco they get with tobacco from another place, then that really weakens the whole argument that their ad worked.

Yes, it is completely possible that they are doing both, but if they are NOT INCREASING it, then we know they aren't substituting one form of tobacco for another. The argument doesn't have to say anything about substitution for us to make that conclusion ourselves.

As far as the correct answer (D) goes, the only real rationale for it being a better answer is relevance. The argument mentions the "tax" portion and answer (D) seems to have more direct relevance to that and that + the fact that it does strengthen the argument makes for a "better" answer. (A), however, does seems to make it stronger, too.

Someone correct me if I'm just delusional.
User avatar
 
tommywallach
Thanks Received: 468
Atticus Finch
Atticus Finch
 
Posts: 1041
Joined: August 11th, 2009
 
 
 

Re: Q12 - One year ago a locak

by tommywallach Sun Mar 01, 2015 9:39 pm

Hey Af,

I wouldn't call you delusional, but I can't say I understand.

If I say: There was an ad that told people to stop eating Twinkies. They stopped eating twinkies. The ad did it.

Then I want to strengthen: People didn't start eating more chocolate chip cookies.

Do you see that the "strengthener" actually has NOTHING to do with the argument? If people stopped eating Twinkies and then IMMEDIATELY started eating cookies, and even if they said "We did this because of that ad that told us to stop eating Ho-Hos!" it still wouldn't matter, because the ad still succeeded in getting people to stop eating Twinkies. So what people then do as a replacement doesn't matter.

The argument is about whether the ads stopped people doing a thing. If they traded it for another thing, we don't actually care. We aren't the FDA or the Surgeon General. We only need to know if they stopped doing the first thing.

You're doing this as a real world question. It isn't a real world question. This is about logic. We want to know if X caused Y. The fact that people didn't start doing Z doesn't actually matter.

-t

P.S. And people DEFINITELY do smoke because of the smoke itself. Most smokers could not simply switch over to chewing tobacco or snorting snuff, as if the things are comparable! And if they did,
Tommy Wallach
Manhattan LSAT Instructor
twallach@manhattanprep.com
Image
 
af10
Thanks Received: 4
Vinny Gambini
Vinny Gambini
 
Posts: 11
Joined: June 30th, 2014
 
 
 

Re: Q12 - One year ago a locak

by af10 Thu Mar 05, 2015 4:18 pm

Hey Tommy,

Thanks for the quick response! I get it now, thank you.

My reason for my initial dissatisfaction about your rationale for (A) was this: I was a little tripped up because you said "this argument is about cigarettes" but by cigarettes, you meant it's actually about smoking. Correct? The actual act of stopping people to physically smoke, right?
User avatar
 
tommywallach
Thanks Received: 468
Atticus Finch
Atticus Finch
 
Posts: 1041
Joined: August 11th, 2009
 
 
 

Re: Q12 - One year ago a locak

by tommywallach Sun Mar 08, 2015 9:56 pm

Aha! Yes indeed. I meant the act of smoking. Good point of distinction. : )

-t
Tommy Wallach
Manhattan LSAT Instructor
twallach@manhattanprep.com
Image
 
LauraS737
Thanks Received: 0
Vinny Gambini
Vinny Gambini
 
Posts: 19
Joined: May 14th, 2017
 
 
 

Re: Q12 - One year ago a locak

by LauraS737 Fri Jun 09, 2017 1:00 am

tommywallach Wrote:
(B) is very tricky. This would actually imply that the effect was larger, but the conclusion says it should be quite small.

-t


Can (B) also be wrong because it's saying that DURING the campaign people still didn't smoke less, and it was only AFTER that people smoke less, which means there could've been an alternative cause as to why people smoke less? I initially translated this answer choice as:

Ad campaign --> No decrease number of smokers.

Please let me know if there is something problematic with this approach.